Warren runs races to support local charities. In April, he ran a 5-kilometer race in 30 minutes to support his local hospital. In October, he will run a 10-kilometer race to support the animal shelter. If he runs at the same rate, how many minutes will it take Warren to run the race in October?

Answers

Answer 1

Warren ran 10 km in 60 minutes or 1 hour.

What is proportionality?

Proportionality is a case when two ratios are equal.

Given that Warren runs for charity, so, he runs for a hospital a 5-kilometer race in 30 minutes, on the same rate he ran a 10-kilometer race to support the animal shelter.

We need to find the time taken by him when he ran 10 km race.

Since, the rate of running is same, that means the speed was equal in both races,

Let t be the required time,

Therefore, using the concept of proportionality,

5 / 30 = 10 / t

5t = 300

t = 60

Therefore, he ran 10 km in 60 minutes or 1 hour.

Learn more about proportionality, click;

https://brainly.com/question/29126727

#SPJ2


Related Questions

Find the ratio of circles to triangles in the diagram below.
ΟΟΔΔΔ
ΟΟΔΔΔ
ΟΟΔΔΔ
Unsimplified ratio of circles to triangles:
try

Answers

Answer:

6:9

Step-by-step explanation:

there are six circle and nine triangle

so the ratio is

6:9

Somebody help (no links please) this is stressing me rn cause this assignment is almost due.

Answers

Step-by-step explanation:

11)

d=√[(-2-(-3))^2+(3-10)^2]=√(-2+3)^2+(-7)^2

=√1+49=√50=7.07 approximately

12)

M (((-1)+(-7))/2 , (2+(-4))/2)=(-8/2 , -2/2)=(-4 ,-1)

(Simplify 6 x m x 5

Answers

Answer:

30m or 6m5

Step-by-step explanation:

If you like my answer than please mark me brainliest thanks

The simplification of the given expression 6 x m x 5 will be 30m.

What is simplification?

Simplification is to make something easier to do or understand and to make something less complicated.

The expression is given below.

6 x m x 5

Then the simplification of the expression will be

30m

Where m can be any number.

Then the expression 6 x m x 5 is written as 30m.

More about the simplification link is given below.

https://brainly.com/question/12616840

#SPJ2

Solve and graph inequality

Answers

Answer:

x ≥ 3 and x ≤ -3      See below

Step-by-step explanation:

| x | ≥ 3

 Case I                                     Case II

x ≥ 3                                          -(x) ≥ 3

                                                  -x ≥ 3  multiply by -1 and change the inequality

                                                  x ≤ -3

The graph:

Graph ≤ or ≥ the closed circle at the endpoint.

  x ≥ 3  closed circle at point 3 and shade to the right .

  x  ≤ -3  closed circle at -3 and shade to the left.

how can you be sure that there are no mistakes on your pay stub assuming that you make $10 an hour and that your employer withholds 10 percent for federal taxes?

Answers

hiStep-by-step explanation:

I need help with this :(please

Answers

Answer:

e Are you need me?you write l donot know.

Solve for x. The triangles in each pair are similar

Answers

9x +1= 0
9x/9 +1-1=0
X= 9-1
X= 8

I will 10 points pls anwser correct

Answers

Answer:

1.5 if you Evaluate and in fraction form [tex]\frac{15}{10}[/tex]

1 4/9 divided by -2/9

Answers

Answer:

6 1/2

Step-by-step explanation:

change 1 4/9 into a improper fraction, which is 13/9, and when dividing fractions flip the second number upside down then multiply. 13/9 times -2/9 is -117/18. simplify, which is 13/2, which is also equal to 6 1/2. hope this helped

Tres socios tienen que repartirse 3000 dólares de beneficios. ¿Cuánto le tocará a cada uno, si el primero tiene que recibir 3 veces más que el segundo y el tercero dos veces más que el primero? Siendo el segundo representado con la letra x.

Answers

Answer:

Abrazos

Step-by-step explanation:

segundo = x

primero = (3*)segundo

tercero = (2*)primero

tercero = 2 * 3 = (6*)segundo

6x + 3x + x = 3000

10x = 3000

x = 300

segundo = 300

primero = 300 * 3 = 900

tercero = 300 * 6 = 1800

checando: 1800 + 900 + 300 = 3000

find an number that is 16 less than twice a number is decrease is 84

Answers

Answer:

50

Step-by-step explanation:

find a number that is 16 less than twice a number is decrease is 84

so if i get this right it is:

84 = 2x-16?

if so we do the following:

84+16 = 2x (Move the variables to one side and the numbers to the other)

100 = 2x (simplify)

50 = x (divide each side)

A 4 in. by 5 in. photo is enlarged so that its width (shorter dimension) is 5 in.

What is the length of the enlarged photo?


5.25 in.


7 in.


6.25 in.


6 in.

Answers

Answer:  6.25 inches (choice C)

========================================================

Explanation:

The jump from 4 inches to 5 inches is 5/4 = 1.25

In other words, 4*1.25 = 5

So we need to multiply every dimension by the scale factor 1.25 to enlarge the image. The 5 inch dimension then goes to 5*1.25 = 6.25 inches

----------

Here's another way you can solve

(old width)/(old length) = (new width)/(new length)

4/5 = 5/x

4x = 5*5

4x = 25

x = 25/4

x = 6.25

Write an equation that describes the nth term of the arithmetic sequence –2, –1.5, –1, –0.5, 0, 0.5, ….

Answers

Answer:

[tex]a_{n}[/tex] = 0.5n - 2.5

Step-by-step explanation:

The nth term of an arithmetic sequence is

[tex]a_{n}[/tex] = a₁ + (n - 1)d

where a₁ is the first term and d the common difference

Here a₁ = - 2 and d = a₂ - a₁ = - 1.5 - (- 2) = - 1.5 + 2 = 0.5 , then

[tex]a_{n}[/tex] = - 2 + 0.5(n - 1) = - 2 + 0.5n - 0.5 = 0.5n - 2.5 ← explicit equation

Which equation is the equation of the line, in point-slope form, that has a slope of - 4 and passes through the point (7, 1)?

Answers

Answer:

y-1=-4(x-7) is the correct answer

Performance task trigonometric identities

Answers

#1

[tex]\\ \sf\longmapsto cos75°[/tex]

[tex]\\ \sf\longmapsto cos(45+30)°[/tex]

[tex]\\ \sf\longmapsto cos45cos30+sin45.sin30[/tex]

[tex]\\ \sf\longmapsto \dfrac{1}{\sqrt{2}}\times \dfrac{\sqrt{3}}{2}+\dfrac{1}{\sqrt{2}}\times \dfrac{1}{2}[/tex]

[tex]\\ \sf\longmapsto \dfrac{\sqrt{3}}{2\sqrt{2}}+\dfrac{1}{2\sqrt{2}}[/tex]

[tex]\\ \sf\longmapsto \dfrac{\sqrt{3}+1}{2\sqrt{2}}[/tex]

#2

[tex]\\ \sf\longmapsto tan\left(-\dfrac{\pi}{12}\right)[/tex]

[tex]\\ \sf\longmapsto tan(-15°)[/tex]

[tex]\\ \sf\longmapsto \dfrac{sin(-15)}{cos(-15)}[/tex]

[tex]\\ \sf\longmapsto \dfrac{-sin15}{cos15}[/tex]

[tex]\\ \sf\longmapsto -\dfrac{\dfrac{\sqrt{3}-1}{2\sqrt{2}}}{\dfrac{\sqrt{3}+1}{2\sqrt{2}}}[/tex]

[tex]\\ \sf\longmapsto -\left(\dfrac{\sqrt{3}-1}{\sqrt{3}+1}\right)[/tex]

Help Please :)
f(x) = 5x +7
and
g(x) = -2x - 4

f(g(x)) = -2x + ????

Answers

Answer:

f(g(x)) =  - 10x - 13

Step-by-step explanation:

Given functions:

f(x) = 5x +7

and

g(x) = -2x - 4

Work out the composite f(g(x)):

f(g(x)) = f( -2x - 4) = 5( - 2x - 4) + 7 = -10x - 20 + 7 = - 10x - 13

Answer:

f(g(x)) = -10x - 13

???? = - 8x - 13

Step-by-step explanation:

f(x) = 5x +7

and

g(x) = -2x - 4

f(g(x)) = f(-2x - 4) = 5(-2x - 4) + 7

f(g(x)) = -10x - 20 + 7

f(g(x)) = -10x - 13

f(g(x)) = -2x - 8x - 13

order the following numbers from least to greatest.
12
V149
11

Answers

Answer:

11 12 v149

thats what I got......

What is the area, in square feet, of a bedroom with l = 912 feet and w = 1114 feet?

Answers

Answer:

The area is 1,015,968

Step-by-step explanation:

Answer:

1,015,968 (otherwise known as a ridiculously large bedroom lol)

Step-by-step explanation:

912 x 1114 = 1,015,968

Do it on any calculator and it will give you the same thing

I don't know if this was a typo because again that is a ridiculously large bedroom so if it is lemme know and I'll edit my answer

What are the x and y-intercepts of the line described by the equation?
2x+4y=12.4
Question 2 options:

x-intercept = -6.2

y-intercept = -3.1


x-intercept = -3.1

y-intercept = -6.2


x-intercept = 6.2

y-intercept = 3.1


x-intercept = 3.1

y-intercept = 6.2



Please help

Answers

X intercept= 6.2
Y intercept= 3.1

A grocery store sells a six pack of bottled water for $3.79, a 9 pack for $4.50, and a 12 pack for $6.98 which package cost the least per bottle explain your reasoning

Answers

Answer:

The six pack of water is the best deal.

Step-by-step explanation:

Divide the number of water bottles per pack by the cost of the pack.

$1.58 per bottle for the six pack.

$2 per bottle for the nine pack.

$1.72 per bottle for the 12 pack.

Answer: The nine pack costs the least.
Explanation:
To figure out cost per bottle, take the price divided by the amount of bottles

$3.79/6 = $0.63 per bottle

$4.50/9 = $0.50

$6.98/12 = $0.58

The nine pack costs the least per bottle at $0.50 per bottle.

I need to know if these answers are correct or not...

1.
x + (x + 1) + (x + 2) + (x + 3) = 70
4x + 6 = 70
4x = 70 - 6
4x = 64
64/4 =
x = 16
16 + 17 + 18 + 19 = 70

10.
Sca = 336/12 = 28 miles/hr
Scu = 336/14 = 24 miles/hr ->
28 = B + C
24 = B - C ->
28= B + C -> B = 28 - C -> 24 = B - C
-> 24 = 28 - C - C
24 -28 = -2C
-4 = -2C
C = 2 -> B = 28 - 2
= 26 miles/hr

Answers

1. Let the four consecutive numbers be x, x+1, x+2, x+3

The sum of four consecutive number is already given to us = 70

Therefore

⇒(x)+(x+1)+(x+2)+(x+3)=70  

We need to combine x as we have four x terms in the equation. The next step is to get all of the x’s on one side of the equation and all the numbers on the other side. The same rule applies – whatever you do to one side of the equation, you must do to the other side as well!

⇒4x+6=70⇒4x=64⇒x=16

So, the four numbers are 16, 17, 18 and 19.

Hence, the greatest number among them is 19.

4xy + 10yᒾ
Tui mún hỏii

Answers

Answer:

4xy+10y²=2y(2x+5y)

Hope it helps!!!

7 4/5 divided by 1 3/10 (please leave explanation)

Answers

Answer:

117/50 or 2 17/50 or 2.34

Step-by-step explanation:

i don't know is just simple

You multiply 7 X5+4 is 39, 39/5 and then multiple

need help asap!!!
(Question:What is 2/3 divided by 4/5?

(Options: 2/3, 5/6, 1 1/5, 3 1/3

Answers

This answer to your equation is 5/6

Answer:

5/6

Step-by-step explanation:

use the quadratic formula to solve the equation 4x^2+x-9=0

Answers

Answer:

see pics

Step-by-step explanation:

dbdksjhddbbdbsksksodhd pls give brainlist

Answer:

x = (-1 ±[tex]\sqrt{145}[/tex] )/8

Step-by-step explanation:

Quadratic formula: x = (-b ± [tex]\sqrt{b^{2} - 4ac}[/tex] )/(2a)  

4x² + x - 9 = 0 ; General form of a quadratic is ax² +bx + c = 0

So a = 4, b = 1, and c = -9

x = -1 ±[tex]\sqrt{1^{2}-4(4)(-9)}[/tex] / 2(4)

x = -1 ± [tex]\sqrt{1 + 144}[/tex] /8

x = -1 ±[tex]\sqrt{145}[/tex] /8

x = (-1 + [tex]\sqrt{145}[/tex])/8    or x = (-1 - [tex]\sqrt{145}[/tex])/8

a rectangle prism has a length of 5 cm, a width of 2.25cm, and a height of 0.5 cm, what is the surface area of the rectangular prisim?

Answers

Answer:

SA = 29.75 cm^2

Question no.8
Plz heeeeelp​

Answers

The exterior angle of each polygon and the adjacent interior angle are

supplementary.

(a) 72°    (b) [tex]\displaystyle 51 \frac{3}{7} ^{\circ}[/tex]    (c) 36°    (d) 20°    (e) 18°

Reasons:

The formula for the exterior angle of a regular polygon is, [tex]\displaystyle \theta = \mathbf{ \frac{360 ^{\circ}}{n}}[/tex]

Where;

θ = The exterior angle of the polygon

n = The number of sides of the regular polygon

(a) The number of sides in a pentagon, n = 5 sides

Therefore, for a regular pentagon, we have;

[tex]Exterior \ angle , \displaystyle \theta = \frac{360 ^{\circ}}{5} = \mathbf{{72^{\circ}}}[/tex]

The measure of the exterior angle of a pentagon, is 72°

(b) An heptagon has n = 7 sides

The measure of the exterior angle, θ, of a regular heptagon is therefore;

[tex]Exterior \ angle \ of \ a \ heptagon , \,\displaystyle \theta = \frac{360 ^{\circ}}{7} = \underline{51 \frac{3}{7} ^{\circ}}[/tex]

(c) A decagon has n = 10 sides, which gives;

[tex]The \ exterior \ angle \ of \ a \regular \ decagon, \,\displaystyle \theta = \frac{360 ^{\circ}}{10} = \underline{36^{\circ}}[/tex]

(d) The exterior angle of an 18-gon with n = 18 sides is given as follows;

[tex]The \ exterior \ angle \ of \ a \ regular \ 18-gon, \,\displaystyle \theta = \frac{360 ^{\circ}}{18} = \underline{20^{\circ}}[/tex]

(e) A regular 20-gon has n = 20 sides

The exterior angle of a regular 20-gon is [tex]\displaystyle \frac{360 ^{\circ}}{20} = \underline{ 18^{\circ}}[/tex]

Learn more about the angles of a polygon here:

https://brainly.com/question/12871621

(((I made up this question so if you can get this you get 51 points// explanation needed prove how to got your answer)))

- multiple choice to make things easier-


Sasha and Frank both have toy cars. Sasha’s car goes 875 meters in 7 minutes. Frank’s car goes 1,232 meters in 11 minutes. Who’s car goes faster and how much faster?

Answers

Answer:

Sasha's car moves 13 meters faster than Frank's car

Step-by-step explanation:

I took Sasha's 875 meters and divided it by 7minutes and got 125

Then, I took Frank's 1232 meters and divided it by 11 minutes and got 112

Finally, I subtracted 125 by 112 and got 13 meters

Answer:

125 × 11 = 1375

1375 > 1232

Sashas car goes faster then Frank's

Sashas car goes 143 meters faster then Franks

Need help with this ASAP!!

Answers

Answer:

64

Step-by-step explanation:

40x1.6

PLEASE HELP AND EXPLAIN

Solve the following equation by factoring 2x^2 - x = 2-4x

Answers

Answer:

x=-2, x=1/2

Step-by-step explanation:

> 2x²-x=2-4x

> 2x²-x+4x-2=0

> 2x²+3x-2=0

> 2x(x+2)-(x+2)=0

> (x+2)(2x-1)=0

> x+2=0; 2x-1=0

> x=-2; x=1/2

Other Questions
Please help first person to answer correctly with no links get 30 points and marked as brainliest no links please I need help with 4-6 Please please help Provide the missing statement and reasons for the following proof: Given: 9(x6)+41=75Prove: x=889Given:9(x-6)+41=75Prove:x=889 what is the answer to 3x-33=3 which amendment set the eligibility requirements for vice president? Agreeing a golden source for a dark item a write about veterinary service available in your locality. consecuencias econmicas ms importante del descubrimiento de Amrica 10+10+10+10-10-10-10-10-10-10-10-10-10-10-9 Anyone knows this? Please I need hellp I can't do itt 2. Lilly and Laura pooled their money to purchasetheir brother, Liam, a video game for his birthday.The video game cost a total of $45. If Laura wasonly able to contribute 2/3 of what Lilly did, howmuch did Laura put toward the present?F. $18G. $21H. $24J. $27K. $30(Retyped) Please help with the answerI just need an answerDon't answer if you don't know so that the points are not wasted King George III would not allow the _______to become an independent nation without a fight. In 1775, the colonies gave them the fight __________was looking for! The first conflicts of the American Revolution took place in the towns of _______________,Massachusetts, on April 19, 1775. What does Mr. Steward mean when he asks ifNorma really "knew [her] husband whats the blank o_er_ap_ng explain the importance of urine analysis in medical diagnosis, giving example PLS help its due tomorrow Ill give brainless What are the duties of a human resources specialist? Choose the answer.Question 5 options:recruit, screen, interview, and place workersplan, develop, and oversee employee compensation programsoversee staff and plan, direct, and coordinate programs to enhance the knowledge and skills of an organization's employeesinterpret and administer labor contracts regarding issues such as wages and salaries, healthcare, pensions, and union and management practices I NEED HELP UNDERSTANDING BOTH OF THESE QUESTIONS if p:q = 1/3:2 and p:r = 1/2:3/4 calculate the ratio p:q:r what is 9/10 minus 3/14?